Tải bản đầy đủ (.doc) (34 trang)

Đa thức bất khả quy

Bạn đang xem bản rút gọn của tài liệu. Xem và tải ngay bản đầy đủ của tài liệu tại đây (278.49 KB, 34 trang )

Đa thức bất khả quy
Lê Xuân Đại – THPT Chuyên Vĩnh Phúc

A. PHẦN MỞ ĐẦU
I. LÍ DO CHỌN ĐỀ TÀI
Các bài toán về đa thức thường xuyên xuất hiện trong các kỳ thi học sinh giỏi
toán và luôn được đánh giá là các bài toán khó. Các bài toán này thường yêu cầu
nghiên cứu tính chất về các hệ số của một đa thức, tính chất về nghiệm của nó hoặc
những bài toán về đa thức nguyên, tính khả quy và được hỏi theo nhiều hình thức
khác nhau.
Bài toán về đa thức với hệ số nguyên và đa thức bất khả quy có vai trò rất
quan trọng và xuất hiện nhiều trong kì thi TST và IMO hàng năm. Thật khó để
khẳng định được một đa thức là bất khả quy hay không. Chuyên đề này muốn khám
phá các vấn đề và các kết quả của đa thức bất khả quy, đưa ra một số tiêu chuẩn và
ví dụ điển hình để áp dụng giải được một lớp bài toán tương tự. Có thể coi những
bài toán về đa thức bất khả quy như là những bài tập lý thuyết, đòi hỏi người học
phải nắm vững từng bước chứng minh như việc chứng minh một định lý toán học
vậy. Đây cũng là lý do mà các bài toán về đa thức bất khả quy luôn gây khó khăn
với học sinh.
Chính vì vậy tác giả quyết định chọn đề tài “Đa thức bất khả quy” , hy
vọng phần nào chia sẻ và giúp các bạn có cách tiếp cận tốt hơn với các bài toán dạng
này.
II. MỤC ĐÍCH VÀ NHIỆM VỤ NGHIÊN CỨU
- Nghiên cứu các dấu hiệu để xét một đa thức có bất khả quy.

1


- Vận dụng tính khả quy và bất khả quy của đa thức vào các bài toán đa thức
nhằm phát huy khả năng tư duy toán học cho học sinh.
- Đề xuất một số biện pháp nhằm rèn luyện, phát huy năng lực tư duy và giải


các bài toán trong phần Đa thức của chương trình chuyên toán THPT.

III. ĐỐI TƯỢNG HỌC SINH
- Đối tượng dạy học của chuyên đề là học sinh chuyên toán của trường THPT
Chuyên, đặc biệt là bồi dưỡng học sinh đội tuyển quốc gia.

2


B. PHẦN NỘI DUNG
I. MỘT SỐ KIẾN THỨC VỀ ĐA THỨC BẤT KHẢ QUY.
1. Đa thức nguyên.
1.1. Định nghĩa. Đa thức với hệ số nguyên là đa thức có dạng
P ( x) = an x n + an−1 x n−1 + ... + a1x + a0 với ai là các số nguyên. Ta ký hiệu tập hợp tất
cả các đa thức với hệ số nguyên là ¢ [ x ] .
1.2. Một số kết quả cơ bản về đa thức với hệ số nguyên.
Cho đa thức P ( x) ∈ ¢ [ x ] .
1.2.1. Nếu P ( x) có nghiệm nguyên x = a thì phân tích được P ( x) = ( x − a)Q( x) với
Q( x) là đa thức với hệ số nguyên.
1.2.2. Nếu a, b nguyên phân biệt thì P (a ) − P( a) chia hết cho a − b .
1.2.3. (Tiêu chuẩn nghiệm). Nếu x =

p
, ( p, q) = 1 là một nghiệm hữu tỷ của P ( x) thì
q

p là ước của a0 và q là ước của an . Đặc biệt nếu an = 1 thì mọi nghiệm hữu tỷ đều là
nghiệm nguyên.
1.2.4. Nếu x = m + n là nghiệm của P ( x) với m, n nguyên, n không chính phương
thì x ' = m − n cũng là nghiệm của P ( x) .

1.2.5. Nếu

x=m+ n

với

m, n

nguyên, n không chính phương thì

P ( x) = M '+ N ' n với M’, N’ nguyên.
Đa thức với hệ số nguyên sẽ nhận giá trị nguyên với mọi giá trị x nguyên. Điều
ngược lại không đúng, có những đa thức nhận giá trị nguyên với mọi x nguyên
nhưng các hệ số của nó không nguyên.
x3 − x
x2 − x
Ví dụ. Các đa thức P ( x) =
và P ( x) =
nhận giá trị nguyên với mọi x
6
2
nguyên.

3


1.3. Đa thức nguyên: Đa thức với hệ số hữu tỷ nhưng nhận giá trị nguyên với mọi x
nguyên được gọi là đa thức nguyên.
Nhận xét. Một đa thức với hệ số hữu tỷ P ( x) bất kỳ có thể biểu diễn dưới dạng
a

Q(x) với a, b là các số nguyên và Q ( x ) là đa thức với hệ số nguyên.
b

2. Đa thức bất khả quy
2.1. Định nghĩa. Cho P ( x) là đa thức với hệ số nguyên. Ta gọi P ( x) là bất khả quy
trên ¢ [ x ] nếu P ( x) không phân tích được thành tích hai đa thức thuộc ¢ [ x ] với bậc
lớn hơn hay bằng 1. Ngược lại thì P ( x) gọi là khả quy trên ¢ [ x ] . Tương tự ta có
định nghĩa đa thức bất khả quy trên ¤ [ x ] ; ¡ [ x ] .
Ví dụ. Đa thức P ( x) = x 2 + 1 bất khả quy trên ¢ [ x ] và trên ¤ [ x ] ; ¡ [ x ] .
2.2. Một số tính chất.
2.2.1. (Quan hệ bất khả quy trên ¢ [ x ] và ¤ [ x ] ): Nếu đa thức P ( x) ∈ ¢ [ x ] bất khả
quy trên ¢ [ x ] thì cũng bất khả quy trên ¤ [ x ] .
Chứng minh.
a) Trước hết ta giới thiệu và chứng minh bổ đề Gauss:
* Ta gọi đa thức P ( x) ∈ ¢ [ x ] là nguyên bản nếu các hệ số của nó nguyên tố cùng
nhau.
* Bổ đề Gauss: Tích của hai đa thức nguyên bản là một đa thức nguyên bản.
Chứng minh bổ đề. Cho hai đa thức nguyên bản P ( x) = an x n + an−1 x n−1 + ... + a1x + a0 ;
Q( x) = bm x m + bm−1 x n−1 + ... + b1 x + b0 thì P ( x).Q( x) = cm+ n x m+ n + ... + c1 x + c0 .
Giả sử P ( x)Q( x) không nguyên bản thì tồn tại một số nguyên tố p là ước chung của
các hệ số c0 , c1 ,..., cm+ n . Vì P ( x) nguyên bản nên gọi i là chỉ số nhỏ nhất mà ai không
chia hết cho p và j là chỉ số nhỏ nhất sao cho b j không chia hết cho p. Khi đó ta thấy
ngay hệ số của x i + j không chia hết cho p, vô lý.
4


b) Chứng minh tính chất 2.2.1. Giả sử P ( x) bất khả quy trên ¢ [ x ] mà P ( x) khả
quy trên ¤ [ x ] . Khi đó P ( x) = P1 ( x).P2 ( x) với P1, P2 là các đa thức bậc nhỏ hơn bậc
của P và có hệ số hữu tỷ.
Đặt P1 ( x) =


a1
a
Q1 ( x), P2 ( x) = 2 Q2 ( x) với (ai , bi ) = 1 và Q1 ( x), Q2 ( x) nguyên bản.
b1
b2

Khi đó P ( x) =

a1a 2
p
Q1 ( x)Q2 ( x) = Q1 ( x)Q2 ( x) với ( p, q ) = 1 . Do P ( x) ∈ ¢ [ x ] nên từ
b1b2
q

đây suy ra các hệ số của Q1 ( x).Q2 ( x) đều chia hết cho q, suy ra Q1 ( x ).Q2 ( x) không
nguyên bản, trái với bổ đề Gauss, mâu thuẫn. Vậy P ( x) bất khả quy trên ¤ [ x ] .
2.2.2. Cho số nguyên a và đa thức P ( x) ∈ ¢ [ x ] . Khi đó P ( x) bất khả quy trên ¢ [ x ]
khi và chỉ khi P ( x + a ) bất khả quy trên ¢ [ x ] .
Chứng minh. Giả sử P ( x) bất khả quy trên ¢ [ x ] và P ( x + a ) lại khả quy trên
¢ [ x ] . Khi đó tồn tại g ( x), h( x) ∈ ¢ [ x ] với bậc lớn hơn hoặc bằng 1 sao cho
P ( x + a ) = g ( x).h( x) . Suy ra P ( x) = g ( x − a).h( x − a) , mâu thuẫn với tính bất khả
quy của P ( x) trên ¢ [ x ] .
2.2.3. Tính khả quy trên ¡ [ x ] : Mọi đa thức có hệ số thực và bậc lớn hơn 2 đều
khả quy trên ¡ .
Thật vậy, dễ thấy các đa thức bậc nhất và bậc hai vô nghiệm trên ¡ thì bất khả quy.
Xét một đa thức P ( x) với bậc lớn hơn 2. Ta có hai trường hợp:
* Nếu P ( x) có nghiệm thực x = α thì hiển nhiên P ( x) khả quy.
* Nếu P ( x) vô nghiệm thực, khi đó P ( x) có bậc chẵn và với mỗi nghiệm phức
x =α


thì

x =α

cũng



nghiệm

Q( x) = ( x − α )( x − α ) = x 2 − 2Re ( α ) x + α

2

của

Khi

đó

là đa thức có hệ số thực và Q( x) là

nhân tử thực sự của P ( x) . Do đó P ( x) khả quy trên ¡ .
5

P ( x) .


Từ đó suy ra mọi đa thức bậc lớn hơn hoặc bằng 1 trong ¡ [ x ] đều phân tích được

một cách duy nhất thành tích các đa thức bậc nhất và đa thức bậc hai vô nghiệm
k

h

2
thực P ( x) = A.∏ ( ai x + bi ) .∏ ( α j x + β j x + γ j ) .
i =1

mi

nj

j =1

Như vậy ta không quan tâm tới tính khả quy của đa thức trên ¡ và £ . Ta chỉ tập
trung vào việc xét tính khả quy và bất khả quy của các đa thức trên ¢ và ¤ .
2.2.4. (Tiêu chuẩn Eisenstein). Dưới đây là tiêu chuẩn quen thuộc và phổ biến để
chứng minh một đa thức bất khả quy.
n
Cho P ( x) = an x + ... + a1 x + a0 ∈ ¢ [ x ] . Giả sử tồn tại số nguyên tố p sao cho

i)

a0 , a1 ,..., an−1 chia hết cho p

ii)

an không chia hết cho p


iii)

a0 không chia hết cho p 2

Khi đó đa thức P ( x) bất khả quy trên ¢ [ x ] .
Chứng minh. Ta đưa ra hai chứng minh cho định lý quan trọng này.
Cách thứ nhất. Giả sử P ( x) khả quy trên ¢ [ x ] , tức là viết được P ( x) = g ( x).h( x) ,
trong

đó

g ( x) = br x r + ... + b1 x + b0 ∈ ¢ [ x ] ; h( x) = cs x s + ... + c1x + c0 ∈ ¢ [ x ]

với

1 ≤ r, s < n .
a0 = b0c0
a = b c + b c
 1 1 0 0 1
Ta có .............
a = b c + b c + .... + b c
k 0
k −1 1
0 k
 k
an = br .cs

(*)

b0 Mp

Theo giả thiết a0 Mp ⇒ 
. Giả sử b0 Mp . Vì a0 không chia hết cho p 2 , nên c0
c0 Mp
không chia hết cho p.

6


Mặt khác các hệ số của g ( x) không thể cùng chia hết cho p (vì an không chia hết
cho p). Khi đó gọi bk là hệ số đầu tiên của g ( x) không chia hết cho p.
Từ (*) và do ak , bk −1 ,..., b0 Mp suy ra bk c0 Mp ⇒ bk Mp , mâu thuẫn.
Từ đó ta suy ra điều phải chứng minh.
Nhận xét. Đây là cách chứng minh đơn giản và thể hiện rõ nhất phương pháp chung
để chứng minh một đa thức bất khả quy. Theo tôi khi giảng dạy nội dung này cho
học sinh cần hết sức chú ý tư tưởng chứng minh, đặc biệt là bước chọn hệ số bk . Với
các bài toán về đa thức bất khả quy thì việc nắm rõ các bước và tư tưởng để chứng
minh một tiêu chuẩn nào đó còn quan trọng hơn cả việc áp dụng tiêu chuẩn đó.
Chẳng hạn bài toán thi chọn đội tuyển Việt nam dự thi IMO 2013 như sau:
"Tìm tất cả các số nguyên dương n > 1 và số nguyên tố p sao cho đa thức
P ( x) = x n − px + p 2 khả quy trên ¢ [ x ] ".
Bài toán này được giải đơn giản như sau nếu biết các bước chứng minh tiêu
chuẩn Eisenstein.
Giả sử P ( x) khả quy trên ¢ [ x ] , tức là viết được P ( x) = g ( x).h( x) , trong đó
g ( x) = br x r + ... + b1 x + b0 ∈ ¢ [ x ] ; h( x) = cs x s + ... + c1x + c0 ∈ ¢ [ x ] với 1 ≤ r , s < n .
a0 = b0c0
a = b c + b c
 1 1 0 0 1
Ta có .............
a = b c + b c + .... + b c
k 0

k −1 1
0 k
 k
an = br .cs

(*)

Từ b0c0 = p 2 , ta xét hai trường hợp:
* Nếu chỉ có b0 hoặc c0 chia hết cho p thì thực hiện tiếp như việc chứng minh tiêu
chuẩn Eisenstein, ta suy ra P ( x) bất khả quy.
* Nếu b0 và c0 cùng chia hết cho p, có thể giả sử b0 = c0 = p . Ta xét tiếp hai trường
hợp nhỏ:
7


- Nếu g và h đều có bậc không nhỏ hơn 2, khi đó ta có − p = b0c1 + b1c0 và
b0c2 + b1c1 + c0b2 = 0 , suy ra b1 + c1 = −1 và b1c1 Mp . Có thể giả sử b1 Mp và c1 không
chia hết cho p. Gọi bk là hệ số đầu tiên của g ( x) không chia hết cho p. Từ hệ thức
ak +1 = bk +1c0 + bk c1 + .... + b0ck +1 , do c0 Mp , c1 không chia hết cho p và theo cách chọn
bk suy ra ak +1 không chia hết cho p, điều này vô lí do k + 1 < n nên ak +1 = 0 .
- Do vậy để P ( x) khả quy thì phải có ít nhất một đa thức g hoặc h có bậc bằng 1.
Nói cách khác thì P ( x) có nghiệm nguyên là x0 .
Khi đó x0 chỉ có thể nhận giá trị là 1; p; −1; − p; p 2 ; − p 2 . Thay vào P ( x) suy ra p
phải chẵn, suy ra p = 2 . Kiểm tra được chỉ có n = 3 thỏa mãn.
Cách thứ 2 (Sử dụng trường ¢ p các số nguyên modulo p): Vẫn giả sử viết được
P ( x) = g ( x).h( x) ,trong đó g ( x), h( x) ∈ ¢ [ x ] với deg g ( x) = r ≥ 1,deg h( x) = s ≥ 1 .
Ta có g ( x)h( x) ≡ P( x) ≡ an x n (mod p ) mà an không chia hết cho p suy ra các hệ số
của lũy thừa bậc cao nhất trong g ( x) và h( x) không chia hết cho p. Do đó tồn tại b
và c sao cho g ( x) ≡ bx r (mod p) và h( x) ≡ cx s (mod p) . Mặt khác do r , s ≥ 1 nên
các hệ số tự do của g ( x) và h( x) chia hết cho p, mẫu thuẫn với giải thiết a0 không

chia hết cho p 2 .
2.2.5. Ta đưa ra tiêu chuẩn bất khả quy thu gọn trên trường ¢ p [ x ] :
n
Giả sử P ( x) = an x + ... + a1 x + a0 ∈ ¢ [ x ] và tồn tại số nguyên tố p sao cho an
n
không chia hết cho p. Nếu đa thức P ( x) = a n x + ... + a1 x + a 0 ∈ ¢ p [ x ] bất khả quy

trên ¢ p [ x ] thì P ( x) bất khả quy trong ¢ [ x ] .
Chứng minh. Giả sử P ( x) khả quy trong ¢ [ x ] , suy ra
P ( x) = an x n + ... + a1 x + a0 = ( bk x k + ... + b1x + b0 ) ( cl x l + ... + c1x + c0 ) (k , l ≥ 1)

(

)(

n
k
l
Khi đó P ( x) = a n x + ... + a1 x + a 0 = b k x + ... + b1x + b 0 c l x + ... + c1 x + c 0

8

)

(*)


Ta có b k .cl = bk .cl = an ≠ 0 vì an không chia hết cho p nên b k ≠ 0, cl ≠ 0 . Như vậy từ
(*) suy ra P ( x) = a n x n + ... + a1 x + a 0 khả quy trên ¢ p [ x ] , mâu thuẫn giả thiết.
Ví dụ: Đa thức P ( x) = 5 x 4 + 7 x3 − 12 x 2 + 4 x − 1 bất khả quy trong ¢ [ x ] . Thật vậy,

xét trong ¢ 2 [ x ] ta có P ( x) = x 4 + x3 + 1 . Nếu P ( x) khả quy trong ¢ 2 [ x ] thì

(

)(

)

P ( x) = x 4 + x3 + 1 = x 2 + ax + 1 x 2 + bx + 1 (1)
Trong (1) cho x = 1 ta được a.b = 1 ⇒ a = b = 1 , thay trở lại (1) ta được điều mâu
thuẫn. Vậy P ( x) bất khả quy trong ¢ 2 [ x ] , tức P ( x) bất khả quy trong ¢ [ x ] .
n
2.2.6. Tiêu chuẩn Eisenstein mở rộng: Cho P ( x) = an x + ... + a1 x + a0 ∈ ¢ [ x ] . Giả

sử tồn tại số nguyên tố p thỏa mãn với một số k ≤ n nào đó:
i) a0 , a1 ,..., ak −1 chia hết cho p
ii) ak không chia hết cho p
iii) a0 không chia hết cho p 2
Thế thì P ( x) có một nhân tử bất khả quy bậc lớn hơn hoặc bằng k (và do đó nếu
không bất khả quy thì sẽ có một nhân tử bậc nhỏ hơn hoặc bằng n − k ).
Với k = n ta được tiêu chuẩn Eisenstein ở trên.
Có thể nói rằng tiêu chuẩn Eisenstein là một trong các tiêu chuẩn cơ bản và
hay được áp dụng nhất để kiểm tra một đa thức bất khả quy. Khi học phần này, nhất
định các em học sinh phải hiểu và nhớ kĩ từng bước chứng minh của tiêu chuẩn
Eisenstein. Nhiều khi bài toán chỉ yêu cầu chứng minh lại tiêu chuẩn trên bằng một
cách hỏi khác đi mà thôi. Xin đưa ra một vài ví dụ cụ thể.
Ví dụ 1. Chứng minh các đa thức sau bất khả quy trên ¢ [ x ]
a) P ( x) = x 5 + 5 x 4 + 20 x 2 + 15

(p=5)


b) P ( x) = 3 x 7 + 7 x 3 + 21x + 28

(p=2)

c) P ( x) = x n − 2 ( n > 1)

(p=2)
9


d) P ( x) = x n − pq , p nguyên tố và ( p, q ) = 1 .
e) P ( x) = x n + 5 x n−1 + 35 (n ≥ 2)

(p=5)

f) P ( x) = x 4 − x3 + 2 x + 1 .
Hint.

Với



dụ

này

ta

chưa


nhận

ra

được

ngay

p.

Ta

tính

P ( x − 1) = x 4 + 3x 3 + 3x 2 + 3x + 3
Với y = x − 1 thì P ( y ) = y 4 + 3 y 3 + 3 y 2 + 3 y + 3 bất khả quy (với p = 3 ). Từ đó
P ( x) bất khả quy.
Ví dụ 2. Cho p nguyên tố lớn hơn 2. Chứng minh đa thức sau bất khả quy trên ¢ [ x ]
P ( x) = x p −1 + x p− 2 + ... + x + 1
x p −1
Hint. Với x khác 1 ta có P ( x) =
.
x −1
Suy ra P ( x + 1) =

( x + 1) p − 1
= x p −1 + C 1p x p− 2 + ... + C pk x p −k −1 + C pp−1
x


1 ≠ 0 (mod p )
 k
Ta có C p ≡ 0(mod p)
, theo tiêu chuẫn Eisenstein thì P ( x + 1) bất khả quy.
 p −1
C p = p ≠ 0 (mod p)
Do đó P ( x) bất khả quy trên ¢ [ x ] (đpcm).
II. MỘT SỐ BÀI TOÁN ĐIỂN HÌNH
Phương pháp chung để chứng minh một đa thức bất khả quy là phương pháp
phản chứng, cụ thể ta giả sử phân tích được đa thức đã cho thành tích hai đa thức với
bậc lớn hơn hay bằng 1 sau đó dẫn đến một điều vô lý. Dù ta có ngay định hướng về
phương pháp như vậy nhưng mỗi bài toán cụ thể lại cần một cách xử lí linh hoạt để
dẫn đến điều mâu thuẫn. Một trong những kĩ thuật hay sử dụng là đánh giá các
nghiệm của đa thức đó hoặc sử dụng trực tiếp một tiêu chuẩn nào đó. Cũng do chủ
yếu là phương pháp phản chứng nên tác giả không chia cụ thể theo phương pháp

10


chứng minh mà chỉ đưa ra một số bài toán điển hình minh họa, sau đó nhấn mạnh
các tư tưởng chính và mở rộng, phát triển bài toán.
Bài toán 1. Cho đa thức P ( x) ∈ ¢ [ x ] bậc 2015 và nhận giá trị bằng 1 hoặc −1 tại
2015 giá trị nguyên khác nhau. Chứng minh rằng P ( x) bất khả quy trên ¢ [ x ] .
Lời giải. Giả sử P ( x) khả quy trên ¢ [ x ] . Đặt P ( x) = g ( x).h( x) , trong đó
g ( x) ∈ ¢ [ x ] ; h( x) ∈ ¢ [ x ] ; 1 ≤ deg h( x) < deg g ( x) < 2015 . Do đó, deg h( x) ≤ 1007
Gọi ai là 2015 số nguyên phân biệt sao cho P (ai ) = ±1 , suy ra g ( ai ).h(ai ) = ±1 .
Gọi p là số giá trị ai mà h(ai ) = 1 , gọi q là số giá trị ai mà h(ai ) = −1. Khi đó
p + q = 2015 . Có thể giả sử p > q ⇒ p ≥ 1008 , do đó đa thức h( x) − 1 có nhiều hơn
1008 nghiệm. Mặt khác deg h( x) ≤ 1007 , suy ra h( x) − 1 ≡ 0 với mọi x, tức là
h( x) ≡ 1 , vô lí. Vậy P ( x) bất khả quy.

Tổng quát hóa
Bài 1.1. Cho đa thức P ( x) ∈ ¢ [ x ] bậc n và nhận giá trị bằng 1 hoặc −1 tại nhiều
n
hơn 2   giá trị nguyên khác nhau. Chứng minh rằng P ( x) bất khả quy trên ¢ [ x ] .
2
Bài 1.2. Cho đa thức P ( x) ∈ ¢ [ x ] bậc n ≥ 8 . Giả sử tồn tại k số nguyên khác nhau
a1 , a2 ,..., ak , k >

n
sao cho P (ai ) = ±1 . Chứng minh rằng P ( x) bất khả quy trên
2

¢ [ x] .
Đặc biệt hóa ta có các bài toán có phát biểu hay và đẹp hơn.
Bài toán 2. Cho n số nguyên ai phân biệt. Chứng minh rằng đa thức
P ( x) = ( x − a1 )( x − a2 )...( x − an ) − 1 bất khả quy trong ¢ [ x ] .
Lời giải. Giả sử P ( x) = g ( x).h( x) , g ( x) ∈ ¢ [ x ] ; h( x) ∈ ¢ [ x ] ;deg g ( x);deg h( x) ≥ 1 .
Suy ra P (ai ) = g ( ai ).h(ai ) = −1 ⇒ g (ai ) + h(ai ) = 0 với mọi i.
11


Như vậy đa thức T ( x) = g ( x) + h( x) có ít nhất n nghiệm và có bậc nhỏ hơn n nên
T ( x) ≡ 0 . Khi đó g ( x) = −h( x) ∀x . Suy ra P ( x) = − [ g ( x) ] 2 .
So sánh hệ số bậc cao nhất ở hai vế ta được điều vô lí ( P ( x) có hệ số bậc cao nhất
bằng 1). Vậy P ( x) bất khả quy.
Bài tương tự
Bài 2.1. Cho n số nguyên ai phân biệt. Chứng minh rằng nếu đa thức
P ( x) = ( x − a1 )( x − a2 )...( x − an ) − 2 ( n ≥ 3) khả quy trên ¢ [ x ] thì n = 3
Bài 2.2. Cho n số nguyên ai phân biệt. Chứng minh rằng nếu đa thức
P ( x) = ( x − a1 )( x − a2 )...( x − an ) + 1 khả quy trên ¢ [ x ] thì P ( x) = x( x − 2) + 1 hoặc

P ( x) = x( x − 1)( x − 2)( x − 3) + 1 .
Tổng quát hơn như sau:
Cho n số nguyên ai phân biệt đôi một nguyên tố cùng nhau. Khi đó đa thức
P ( x) = ( x − a1 )( x − a2 )...( x − an ) + 1 bất khả quy trên ¢ [ x ] trừ các trường hợp sau
+) P ( x) = ( x − a)( x − a − 2) + 1 = ( x − a − 1) .
2

+) P ( x) = ( x − a)( x − a − 1)( x − a − 2)( x − a − 3) + 1 = ( x − a − 1) ( x − a − 2 ) + 1 .
2

Bài toán 3. Cho đa thức P ( x) thỏa mãn xP( x − 1) = ( x − 2014).P( x) và
P (2014) = 2014! . Chứng minh rằng đa thức f ( x) = P 2 ( x) + 1 bất khả quy trong
¢ [ x] .
Lời giải. Trước tiên tìm được P ( x) = x( x − 1)( x − 1)...( x − 2013) .Giả sử f(x) khả quy
trong ¢ [ x ] :
f ( x) = g ( x).h( x) , trong đó g ( x) ∈ ¢ [ x ] ; h( x) ∈ ¢ [ x ] ;deg g ( x);deg h( x) ≥ 1 .
Vì f ( x) vô nghiệm nên g ( x), h( x) cũng vô nghiệm. Không mất tính tổng quát, giả
sử g ( x) > 0; h( x) > 0 với mọi x và g ( x), h( x) có hệ số cao nhất bằng 1.
Ta có f (i ) = g (i ).h(i ) = 1 ⇒ g (i ) = h(i ) = 1 với mọi i = 0,2013 .
12


Giả sử deg g ( x) = m;deg h( x) = n (n ≥ m)
Nếu m < n , do m + n = 2.2014 , suy ra m < 2014 .
Như vậy đa thức g ( x) − 1 có bậc m < 2014 và lại có ít nhất 2014 nghiệm nên
g ( x) ≡ 1 . Đây là điều mâu thuẫn vì deg g ( x) ≥ 1 . Vậy m = n = 2014 . Khi đó
g ( x) = h( x) = x( x − 1)( x − 1)...( x − 2013) + 1 = P( x) + 1 .
Do đó f ( x) = h 2 ( x) = [ P( x) + 1] = P 2 ( x) + 1 ⇒ P ( x) = 0 , vô lí.
2


Vậy f ( x) bất khả quy trong ¢ [ x ] .
Bài toán trên chỉ là trường hợp đặc biệt của bài toán tổng quát sau:
Bài 3.1. Cho n số nguyên ai

phân biệt. Chứng minh rằng đa thức

f ( x) = ( x − a1 ) 2 ( x − a2 ) 2 ...( x − an ) 2 + 1 bất khả quy trong ¢ [ x ] .
Lời giải. Giả sử f ( x) = g ( x).h( x) , g ( x) ∈ ¢ [ x ] ; h( x) ∈ ¢ [ x ] ;deg g ( x);deg h( x) ≥ 1 .
Do f ( x) > 0 với mọi x nên có thể giả sử g ( x) > 0; h( x) > 0 với mọi x.
Ta có f ( ai ) = g (ai ).h(ai ) = 1 ⇒ g ( ai ) = h( ai ) = 1 với mọi i = 1, n
Từ đó thấy ngay deg g ( x) = deg h( x) = n .
Như vậy g ( x) − 1 = A ( x − a1 ) ( x − a2 ) ..( x − an ) ; h( x ) − 1 = B ( x − a1 ) ( x − a2 ) ..( x − an )
Khi đó
f ( x) = ( x − a1 ) 2 ( x − a2 ) 2 ...( x − an ) 2 + 1
= 1 + A ( x − a1 ) ...( x − an )  1 + B ( x − a1 ) ...( x − an ) 
So sánh hệ số của x 2n và hệ số tự do ta được:
AB = 1 và a12 a22 ...an2 = ( A + B )(−1) n .a1a2 ...an + AB.a12 a22 ...an2

Suy ra ( A + B )(−1) n .a1a2 ...an = 0 ⇒ A + B = 0 , mâu thuẫn với AB = 1
Vậy f ( x) bất khả quy trong ¢ [ x ] .

13


Bài 3.2: Cho n và k là số nguyên dương và a1 , a2 ,..., an là các số nguyên phân biệt.
k

k

k


Chứng minh rằng đa thức f ( x) = ( x − a1 ) 2 ( x − a2 ) 2 ...( x − an ) 2 + 1 bất khả quy trong
¢ [ x] .
Bài toán 4. Cho a, n nguyên và p là số nguyên tố thỏa mãn p > a + 1 . Chứng minh
rằng đa thức f ( x) = x n + ax + p bất khả quy trong ¢ [ x ] .
Lời giải. Giả sử f ( x) = g ( x).h( x) , g ( x) ∈ ¢ [ x ] ; h( x) ∈ ¢ [ x ] ;deg g ( x);deg h( x) ≥ 1 .
 g (0) = ± p; h(0) = ±1
Suy ra p = f (0) = g (0).h(0) ⇒ 
 g (0) = ±1; h(0) = ± p
Giả sử g (0) = ±1 ⇒ g ( x) = ± x m + ... ± 1 .
Gọi z1 , z2 ,..., zm là các nghiệm của g ( x) (thực hoặc phức)
Ta có 1 = g (0) = z1 . z2 ... zm , suy ra tồn tại zi ≤ 1 .
n

n
n
Ta có f ( zi ) = 0 ⇒ p = − zi − azi ⇒ p = − zi − azi ≤ zi + a . zi ≤ 1 + a , mâu thuẫn

với giả thiết. Vậy f ( x) bất khả quy trong ¢ [ x ] .
Bài toán tổng quát:
n
Bài 4.1. Cho đa thức P ( x) = an x + ... + a1 x + a0 ∈ ¢ [ x ] thỏa mãn a0 là số nguyên tố

và a0 > a1 + a2 + ... + an . Chứng minh rằng P ( x) bất khả quy trong ¢ [ x ] .
Lời giải. Gọi α là một nghiệm phức của P ( x) . Giả sử α ≤ 1
n
Ta có a0 = anα + ... + a1α ≤ a1 + a2 + ... + an , mâu thuẫn với giả thiết.

Từ đó suy ra α > 1 với mọi nghiệm α của P ( x) (1)
* Giả sử P ( x) khả quy, suy ra P ( x) = g ( x).h( x) , trong đó

g ( x) ∈ ¢ [ x ] ; h( x) ∈ ¢ [ x ] ;deg g ( x);deg h( x) ≥ 1
 g (0) = 1
Suy ra a0 = g (0).h(0) ⇒ 
 h(0) = 1
14


Có thể giả sử g (0) = 1 . Gọi b là hệ số bậc cao nhất của g ( x) và gọi z1 , z2 ,..., zk là
các nghiệm của g ( x) (thực hoặc phức) thì z1.z2 .....zk =

1 1
= ≤ 1.
b b

Đương nhiên z1 , z2 ,..., zk là các nghiệm của P ( x) nên điều này mâu thuẫn với (1).
Vậy P ( x) bất khả quy trong ¢ [ x ] .
n 1
Nhận xét 1. Đa thức f ( x) bậc n bất khả quy trên ¤ [ x ] khi và chỉ khi x f  ÷ bất
x

khả quy trên ¤ [ x ] và f ( x) khả quy trên ¤ [ x ] khi và chỉ khi f ( x) khả quy trên
¢ [ x ] . Từ đó có thể ra một bài toán tương tự bài 4.1 trên nhưng ở dạng ẩn, khó phát
hiện bản chất hơn như sau:
m n
n −1
Bài 4.2. Cho đa thức f ( x) = p x + an−1x + ... + a1x + a0 ∈ ¢ [ x ] , với n ≥ 2; m ≥ 1 và

a0 , an−1 ≠ 0 , p là số nguyên tố và an−1 không chia hết cho p. Chứng minh rằng nếu
p m > a0 + a1 + ... + an−1 thì f ( x) bất khả quy trên ¤ [ x ] .
n 1

n
n −1
m
Lời giải. Xét P ( x) = x f  ÷ = a0 x + a1 x + ... + an−1x + p .
x

Vẫn giả sử P ( x) = g ( x).h( x) , g ( x) ∈ ¢ [ x ] ; h( x) ∈ ¢ [ x ] ;deg g ( x);deg h( x) ≥ 1 .
Từ an−1 không chia hết cho p ta thấy không thể có đồng thời g (0)Mp và h(0)Mp . Mà
g (0) h(0) = p m . Do vậy phải có g (0) = 1 hoặc h(0) = 1 . Giả sử g (0) = 1 .
Tương tự bài 4.1 thì mọi nghiệm của P ( x) đều thỏa mãn z > 1 .
Gọi z1 , z2 ,..., zk là các nghiệm của g ( x) thì nó là nghiệm của P ( x) ⇒ zi > 1 .
Vậy 1 = g (0) = z1.z2..... zk > 1 , vô lí. Bài toán được chứng minh hoàn toàn.
Nhận xét 2. Từ kết quả tổng quát này ta có thể thay đổi một cách hỏi khác để được
một bài toán mới khó hơn như sau:

15


Bài 4.3. Cho đa thức P ( x) ∈ ¢ [ x ] . Chứng minh rằng có vô số số nguyên k mà
P ( x) + k là đa thức bất khả quy.
n
Lời giải. Giả sử P ( x) = an x + ... + a1 x + a0 ∈ ¢ [ x ] . Ta chọn k nguyên dương sao cho

n

k + a0 nguyên tố và k + a0 > ∑ ai , khi đó theo kết quả bài toán tổng quát thì
i =1

P ( x) + k bất khả quy.
Bài 4.4. Cho đa thức f ( x) ∈ ¢ [ x ] có các nghiệm z1 , z2 ,..., zn . Đặt M = max zi .

Chứng minh rằng nếu f ( x0 ) nguyên tố với mọi số nguyên x0 và x0 > M + 1 thì
f ( x) bất khả quy trong ¢ [ x ] .
Tổng quát hơn là định lý Perron sau đây
n
n −1
Tiêu chuẩn Perron: Cho đa thức P ( x) = x + an−1 x + ... + a1 x + a0 ∈ ¢ [ x ] ; a0 ≠ 0

thỏa mãn an−1 > 1 + a0 + a1 + ... + an−2 . Chứng minh rằng P ( x) bất khả quy trong
¢ [ x] .
Chứng minh.
Ta cần một Bổ đề sau:
n
Cho P ( x) = x + ... + a1 x + a0 ∈ ¢ [ x ] với an−1 > 1 + a0 + a1 + ... + an−2 . Khi đó

có đúng một nghiệm của P ( x) thỏa mãn z > 1 và (n − 1) nghiệm (phức) còn lại
thỏa mãn z < 1.
Việc chứng minh bổ đã có trong nhiều tài liệu, xin dành cho bạn đọc.
Trở lại việc chứng minh định lý:
Giả sử P ( x) = g ( x).h( x) , g ( x) ∈ ¢ [ x ] ; h( x) ∈ ¢ [ x ] ;deg g ( x);deg h( x) ≥ 1 .
Theo bổ đề thì P ( x) chỉ có đúng một nghiệm thỏa mãn z0 > 1, có thể giả sử z0 là
nghiệm của h( x) khi đó thì tất cả các nghiệm của g ( x) đều có modun nhỏ hơn 1.
Gọi z1 , z2 ,..., zk là các nghiệm của g ( x) ⇒ zi < 1 , suy ra g (0) = z1.z2 ...zk < 1 .
16


Mặt khác P (0) = g (0) . h(0) = a0 ≥ 1 ⇒ g (0) ≥ 1 , mâu thuẫn.
Sau đây là ba ví dụ cho việc áp dụng tiêu chuẩn Perron.
Bài 4.5. Cho P ( x) ∈ ¢ [ x ] . Chứng minh rằng tồn tại số nguyên dương N sao cho với
mọi k ≥ N thì x 2 P ( x) + kx + 1 bất khả quy.
Lời giải. Với P ( x) ∈ ¢ [ x ] ;deg P ( x) = n và

P ( x) = f ( x).g ( x); f ( x), g ( x) ∈ ¢ [ x ] ;deg f = r ≥ 1,deg g = n − r ≥ 1
 k
n 1
Khi đó x P  ÷ =  x . f
 x 

 1    n−k  1  
 ÷ .  x .g  ÷ . Như vậy P ( x) khả quy khi và chỉ khi
 x  
 x 

1
x n P  ÷ bất khả quy.
 x
1 
1 k
n+2  1
Rõ ràng với k đủ lớn thì đa thức h( x) = x  2 . f  ÷+ + n+2  thỏa mãn các
 x x x 
x
điều kiện của định lý Perron nên h( x) bất khả quy, ta có đpcm.
Bài 4.6. (Iran TST 2003). Cho các đa thức f1 , f 2 ,,, f n ∈ ¢ [ x ] . Chứng minh rằng tồn
tại một đa thức khả quy

g ( x) ∈ ¢ [ x ]

sao cho tất cả các đa thức

f1 + g , f 2 + g ,..., f n + g đều bất khả quy.
Thật vậy, với M đủ lớn và số m > max { deg f1 ,deg f 2 ,...,deg f n } thì theo tiêu

chuẩn Perron đa thức Q( x) = x m+1 + M .x m + f i ( x) là bất khả quy. Do đó có thể chọn
g ( x) = x m+1 + M .x m thỏa mãn bài toán.
Bài 4.7. Gọi ( f n ) là dãy Fibonacci, các đa thức f 0 = 0; f1 = 1; f n+1 = f n + f n−1 . Chứng
minh rằng với mỗi n ≥ 2 đa thức P ( x) = x n + f n f n+1 x n−1 + ... + f 2 f 3 x + f1 f 2 bất khả
quy trong ¢ [ x ] .
Thật vậy, theo tiêu chuẩn Perron ta chỉ cần chứng minh:
f n+1 f n > f n f n−1 + ... + f 2 f1 + 1 với mọi n ≥ 3 (1)
17


Ta chứng minh (1) bằng quy nạp theo n. Với n = 3 thì dễ thấy (1) đúng.
Giả sử (1) đúng đến n, ta có:
f n+1 f n + f n f n −1 + ... + f 2 f1 + 1 < f n +1 f n + f n +1 f n < f n+2 f n+1 , vì 2 f n < f n+ 2 = f n + f n +1
Vậy (1) đúng với mọi n ≥ 3 , bài toán được chứng minh.
Bài toán 5 (IMO 1993). Chứng minh rằng đa thức sau bất khả quy trong ¢ [ x ] .
f ( x) = x n + 5 x n−1 + 3 ( n ∈ ¢ + , n > 1)
Lời giải. Giả sử f ( x) khả quy, suy ra f ( x ) = g ( x).h( x ) , với
g ( x) = as x s + ... + a1 x + a0 ∈ ¢ [ x ] ; h( x) = bt x t + ... + b1 x + b0 ∈ ¢ [ x ] ,
s + t = n, as = bt = 1 và s, t ≥ 1 .
3 thì b0 không chia hết cho 3.
Khi đó a0b0 = 3 , có thể giả sử a0 M
Gọi k là số nguyên dương nhỏ nhất mà ak không chia hết cho 3.
Từ k ≤ n − 1 suy ra hệ số của x k trong f ( x) chỉ bằng 0 hoặc 5.
*

Nếu

hệ

số


của

xk

trong

f ( x) bằng

0

thì

k < n −1



a0bk + a1bk −1 + ... + ak −1b1 + ak b0 = 0
Từ 3 | ai , ∀i = 0, k − 1 ta có 3 | ak b0 ⇒ 3 | ak , vô lí.
* Nếu hệ số của x k trong f ( x) bằng 5 thì k = n − 1 . Khi đó h( x) = 1 + x và
ai = 3ai ', ∀i = 1, n − 2 .
2
'
n −1
n
Do đó f ( x) = 3 + 3 ( a1 '+ 1) x + 3 ( a2 '+ a1 ' ) x + ... + ( 3an−2 + 1) x + x

Suy ra ai ' ≡ ±1 (mod5), ∀i = 1, n − 2 , như vậy 5 = 3an' −2 + 1 không chia hết cho 5, vô
lí. Vậy f ( x) bất khả quy trong ¢ [ x ] .
Bài toán 6. Cho a, m, n nguyên dương và p là số nguyên tố thỏa mãn p < a − 1 .

n
Chứng minh rằng đa thức f ( x) = x m ( x − a ) + p bất khả quy trong ¢ [ x ] .

Lời giải. Giả sử f ( x) = g ( x).h( x) , g ( x) ∈ ¢ [ x ] ; h( x) ∈ ¢ [ x ] ;deg g ( x);deg h( x) ≥ 1 .
Suy ra p = f (0) = g (0).h(0) ⇒ g (0) = 1 hoặc h(0) = 1 .
18


Giả sử g (0) = 1 , gọi z1 , z2 ,..., zk là các nghiệm của g(x) (thực hoặc phức), khi đó
g ( x) = ( x − z1 ) ( x − z2 ) ...( x − zk ) và 1 = g (0) = z1 . z2 ... zk , nên tồn tại zi ≤ 1
Ta có f ( zi ) = 0 ⇒ zim ( zi − a ) = − p, ∀i = 1, k .
n

Suy ra g (a ) = ( z1 − a ) ...( zk − a ) = p k . Mà g ( a) | p ⇒ g (a) = p và n = k
n

n

Tuy nhiên a | g (a) − g (0) = p ± 1 , vô lí vì p < a − 1 . Vậy f ( x) bất khả quy.
Bài toán tương tự
Bài 6.1. Cho p là số nguyên tố lẻ và đa thức f ( x) = x p − x + p
a) Chứng minh rằng tất cả các nghiệm của f ( x) đều có mođun nhỏ hơn

p −1

p.

b) Chứng minh rằng f ( x) bất khả quy trong ¢ [ x ] .
Bài 6.2. Cho p là số nguyên tố và a nguyên dương không chia hết cho p. Chứng
minh rằng đa thức f ( x) = x p − x − a bất khả quy trong ¢ [ x ] .

Với bài toán này có thể đưa ra một chứng minh khá đơn giản khi sử dụng trường ¢ p
Thật vậy, chú ý rằng nếu f ( x) khả quy trên ¢ [ x ] thì cũng khả quy trên ¢ p [ x ] .
Giả sử f ( x) = g ( x).h( x) , trong đó g ( x) ∈ ¢ [ x ] ; h( x) ∈ ¢ [ x ] ;1 ≤ deg g ≤ p − 1 .
p
Xét b ∈ ¢ p , ta có g ( x − b)h( x − b) = ( x − b ) − ( x − b) − a ≡ x p − x − a

Do đó f ( x) chia hết cho p đa thức dạng gi ( x) = g ( x − i ); i = 0, p − 1 .
k
Vì deg g ≤ p − 1 nên từ đẳng thức ( x − i ) − ( x − j ) k = ( j − i ).k .x k −1 + ... suy ra tất cả
p
các đa thức gi ( x) phân biệt. Do đó p = deg ( x − x − a ) ≥ p.deg g ( x) ⇒ deg g = 1 .

Mà a không chia hết cho p nên

f ( x) không có nghiệm trên ¢ p vì

b p − b = 0, ∀b ∈ ¢ p , mâu thuẫn (đpcm).
Bài toán 7 (Nhật bản 1999). Chứng minh rằng đa thức sau bất khả quy
f ( x) = ( x 2 + 12 )( x 2 + 22 )...( x 2 + n 2 ) + 1
19


Lời giải. Giả sử f ( x) = g ( x).h( x) , g ( x) ∈ ¢ [ x ] ; h( x) ∈ ¢ [ x ] ;deg g ( x);deg h( x) ≥ 1 .
Ta có f ( ± k .i ) = 1 với mọi k = 1, n , ở đó i 2 = −1. Suy ra 1 = g ( ki ).h( ki ) với mọi
k ∈ { ±1; ±2;...; ± n} .
Do chỉ có 4 cách phân tích số 1 thành tích của các số nguyên trong ¢ [ i ] là
1 = 1.1 = (−1).(−1) = i.(−i) = ( −i ).i

nên


với

( g (ki); h(ki) ) ∈ { (1;1);(−1; −1);(i; −i);(−i; i)} .

mỗi

k ∈ { ±1; ±2;...; ± n}

thì

Trong cả 4 trường hợp ta đều có

( )

g ( ki ) = h( ki) = h ki = h ( −k .i ) . Suy ra đa thức g ( x) − h(− x) có bậc nhỏ hơn 2n mà
lại có 2n nghiệm phân biệt, do đó g ( x) − h(− x) ≡ 0 ∀x ∈ ¡ ⇒ g ( x) ≡ h( − x) .
Đặc biệt g (0) ≡ h(0) ⇒ f (0) = g 2 (0) là số chính phương, hay ( n!) + 1 là số chính
2

phương, vô lí. Vậy f ( x) bất khả quy trong ¢ [ x ] .
Đây từng là một bài toán kinh điển và rất hay về đa thức bất khả quy, lời giải
bài toán đã sử dụng triệt để tính chất nghiệm (phức) của một đa thức đó là: "Một đa
thức bậc n luôn có n nghiệm phức". Việc sử lý linh hoạt đơn vị ảo i (i 2 = −1) là một
nghệ thuật và mang lại nét đẹp và tính kinh điển của bài toán này.
Bài toán trên chỉ là một trường hợp riêng của bài toán sau:
Bài 7.1. (Romanian TST 2003). Cho đa thức f ( x) ∈ ¢ [ x ] monic, bất khả quy và
f (0) không là số chính phương. Chứng minh rằng g ( x) = f ( x 2 ) cũng là đa thức
bất khả quy.
Có thể đưa ra một lời giải cho bài toán này như sau:
Giả sử g ( x) = h( x).k ( x); h( x), k ( x ) ∈ ¢ [ x ] ;deg h,deg k ≥ 1 . Ta có

f ( x) = h

( x ) .k ( x ) = h ( x) +
1

x .h2 ( x)   k1 ( x ) + x .k 2 ( x )  ,

trong đó h1 , h2 , k1 , k2 ∈ ¢ [ x ] .
Ta có h2 ( x).k1 ( x) + h1 ( x).k2 ( x) = 0 nên tồn tại các đa thức p, q, r , s ∈ ¢ [ x ] sao cho:
20


h2 = p.q; k1 = r.s; h1 = p.r ; k2 = −q.s . Khi đó
f ( x) = p( x)  r ( x) + x .q( x)  .s ( x) r ( x) − x .q( x) 
= p( x) s ( x)  r 2 ( x) − x.q 2 ( x) 
2
2
Chú ý rằng deg ( r ( x) − x.q ( x) ) ≥ 1 và f ( x) monic, bất khả quy suy ra

p ( x) s ( x) = ±1 . Do đó f ( x) = r 2 ( x) − x.q 2 ( x) và đặc biệt f (0) = r (0) 2 là số chính
phương, mâu thuẫn. Vậy g ( x) = f ( x 2 ) bất khả quy.
Chú ý: Nếu đa thức f ( x) bất khả quy thì không thể suy ra các đa thức f ( x 2 ), f ( x 3 )
bất khả quy. Chẳng hạn, xét f ( x) = x 2 + x + 1 bất khả quy nhưng
f ( x 2 ) = x 4 + x 2 + 1 = ( x 2 + x + 1) ( x 2 − x + 1) là đa thức khả quy.

Bài toán 8 (VMO 2014). Cho đa thức P ( x) = ( x 2 − 7 x + 6) 2 n + 13 với n là số nguyên
dương. Chứng minh rằng P ( x) không thể biểu diễn được dưới dạng tích của n + 1
đa thức khác hằng số với hệ số nguyên.
Lời giải. Hướng tiếp cận tự nhiên nhất vẫn là phản chứng. Giả sử
P ( x) = P1 ( x).P2 ( x)...Pn+1 ( x) với Pi ( x) ∈ ¢ [ x ] và deg Pi ( x) ≥ 1 .

Do P ( x) vô nghiệm thực nên các Pi ( x) phải có bậc chẵn. Vì tổng các bậc của Pi ( x)
bằng 4n nên phải có ít nhất hai đa thức chẳng hạn P1 ( x), P2 ( x) có bậc bằng 2.
Do

P ( x)

monic

nên

P1 ( x), P2 ( x)

P1 ( x) = x 2 + ax + b; P2 ( x) = x 2 + cx + d





níc,

P1 ( x), P2 ( x)




thể

giả

sử


nghiệm

thực

nên

P1 ( x) > 0, P2 ( x) > 0 ∀x ∈ ¡ .
Ta có 13 = P(1) = P1 (1).P2 (1)...Pk (1) và 13 = P(6) = P1 (6).P2 (6)...Pk (6) .
Giả sử P1 (1) = 1 ⇒ a = −b . Khi đó P1 (6) = 36 − 5b
Ta thấy P1 (6) = 36 − 5b > 0 và không thể là 13 nên P1 (6) = 36 − 5b = 1

21


Từ đó tìm ra a = −7; b = 7 . Khi đó P1 ( x) = x 2 − 7 x + 7 lại có nghiệm thực, mâu
thuẫn. Vậy bài toán được chứng minh.
Nhận xét 1. Phát biểu của bài toán mới đầu có cảm giác khó khăn và làm cho các
học sinh e ngại. Nhưng hóa ra bản chất của điều kiện P ( x) không phân tích được
thành tích của n+1 đa thức chỉ là để suy ra trong đó có ít nhất một (hay đúng ra là
hai) đa thức có bậc 2.
Ta có thể chứng minh kết quả mạnh hơn: Nếu P ( x) = ( x 2 − 7 x + 6) 2 n + 13 có thể
phân tích thành tích của hai đa thức Q ( x), S ( x) khác hằng với hệ số nguyên thì
Q( x), S ( x) đều có bậc bằng 2n.
Thật vậy, gọi x1 , x2 ,..., x4 n là các nghiệm phức của P ( x)
Giả sử Q( x) = ( x − x1 )( x − x2 )...( x − xk );1 ≤ k < 4n
Ta có [ ( xi − 1)( xi − 6) ]

2n


= −13 ⇒ ( xi − 1)( xi − 6) = 131/2 n (1)

Mặt khác Q(1) = (1 − x1 )(1 − x2 )...(1 − xk ) nguyên nên (1 − x1 )(1 − x2 )...(1 − xk ) nguyên
Tương tự (6 − x1 )(6 − x2 )...(6 − xk ) nguyên.
Do đó ( x1 − 1)( x1 − 6)( x2 − 1)( x2 − 6)...( xk − 1)( xk − 6) là số nguyên.
Khi đó theo (1) thì 13k /2 n nguyên, suy ra k = 2n , điều phải chứng minh.
Từ kết quả này nếu n > 1 thì ta suy ra ngay kết quả bài toán, còn nếu n=1 thì kiểm
tra được P ( x) bất khả quy.
Nhận xét 2. Thực ra từ chứng minh kết quả trên ta có thể suy ra P ( x) bất khả quy
với mọi n nguyên dương. Thật vây, ta có
Q(1)Q(6) = ( x1 − 1)( x1 − 6)( x2 − 1)( x2 − 6)...( xk − 1)( xk − 6) = 13k /2 n = 13 (2)
Nhưng 5 | Q(6) − Q(1) nên dễ thấy không có đa thức Q( x) thỏa mãn. Vậy P ( x) bất
khả quy.
Bài tương tự: Tìm số nguyên dương n nhỏ nhất sao cho đa thức P ( x) = x n−4 + 4n có
thể phân tích thành tích của 4 đa thức bậc lớn hơn hoặc bằng 1 với hệ số nguyên.
22


Bài toán 9. Cho đa thức f ( x) ∈ ¢ [ x ] bậc n. Nếu tồn tại ít nhất 2n+1 số nguyên phân
biệt m sao cho f (m) nguyên tố thì f ( x) bất khả quy.
Lời giải. Giả sử f ( x) khả quy suy ra f ( x ) = g ( x).h( x ) , trong đó
g ( x) ∈ ¢ [ x ] ; h( x) ∈ ¢ [ x ] ; r = deg g ( x); s = deg h( x ); r , s ≥ 1 và n = r + s .
Ta có f (m) = g (m) . h(m) là số nguyên tố nên g ( m) = ±1 hoặc h(m) = ±1
Do deg g ( x) = r nên mỗi đa thức g ( x) + 1 và g ( x) − 1 có nhiều nhất r nghiệm. Suy
ra có nhiều nhất r số nguyên m để g (m) = 1 và nhiều nhất r số nguyên m để
g ( m) = −1 . Tức là có nhiều nhất 2r số nguyên m để g ( m) = ±1 .
Tương tự có nhiều nhất 2s số nguyên m để h(m) = ±1 . Như vậy có nhiều nhất
2r + 2 s = 2n số nguyên m để f (m) là số nguyên tố, mâu thuẫn với giả thiết.
Vậy f ( x) bất khả quy trên ¢ [ x ] .
Có thể chứng minh kết quả chặt hơn nữa như sau:

Cho đa thức f ( x) ∈ ¢ [ x ] bậc n. Nếu tồn tại ít nhất n+5 số nguyên phân biệt m
sao cho f (m) nguyên tố thì f ( x) bất khả quy.
Bài toán 10. Cho p là số nguyên tố và an an−1...a1a0 là biểu diễn thập phân của nó (
an > 1 ). Chứng minh rằng P ( x) = an x n + ... + a1 x + a0 bất khả quy.
Lời giải. Giả sử P ( x) = g ( x).h( x) , trong đó g ( x) ∈ ¢ [ x ] ; h( x) ∈ ¢ [ x ] .
Gọi x1 , x2 ,..., xk là các nghiệm của g ( x) và xk +1 , xk + 2 ,..., xn là các nghiệm của h( x) .
Theo giả thiết ta có P (10) = g (10).h(10) là số nguyên tố, nên có thể giả sử
g (10) = 1 ⇒ (10 − x1 )(10 − x2 )...(10 − xk ) = 1 (1).
Tuy nhiên theo kết quả của bài toán biên của nghiệm ta có xi < 1 +

9 11
= <9
2 2

suy ra 10 − xi > 1 ∀i , mâu thuẫn với (1). Vậy P ( x) bất khả quy.
Bài toán này chỉ là trường hợp đặc biệt của Tiêu chuẩn Cohn's sau đây
23


Tiêu chuẩn Cohn's: Cho p là số nguyên tố và an an−1...a1a0 là biểu diễn của p trong
cơ số b ( b ≥ 2 ). Chứng minh rằng P ( x) = an x n + ... + a1 x + a0 bất khả quy.
Việc chứng minh tiêu chuẩn này khá phức tạp và dựa vào 2 bổ đề sau:
n
Bổ đề 1: Cho đa thức P ( x) = an x + ... + a1 x + a0 ∈ ¢ [ x ] thỏa mãn an ≥ 1; an−1 ≥ 0 và

ai ≤ H , ∀i = 0,1,..., n − 2 (H là một hằng số dương). Khi đó với mỗi nghiệm phức α
1 + 1 + 4H
của P ( x) thì hoặc Re α ≤ 0 hoặc α <
.
2

Chứng minh bổ đề 1: Nếu z > 1 và Re z > 0 , ta có
 1
f ( z)
an−1
1 
a 
H


a
+

H
+
...
+
> Re  an + n −1 ÷− 2

n
2
n ÷
n
 z
z
z
z  z −z

z ÷



≥1−
Suy ra z ≥

H
2

z −z

2

=

z − z −H
2

z −z

≥ 0.

1 + 1 + 4H
.
2

Như vậy nếu α ≥

1 + 1 + 4H
và Reα > 0 thì α không thể là nghiệm của P ( x) , bổ
2

đề được chứng minh.

n
n −1
Bổ đề 2: Cho đa thức P ( x) = x + an−1 x ... + a1x + a0 ; ai ∈ { 0;1} . Khi đó tất cả các

nghiệm của P ( x) nằm trong nửa mặt phẳng Re z <

3
.
2

Chứng minh bổ đề 2. Trường hợp n = 1,2 là đơn giản.
Xét n ≥ 3 , khi đó với z ≠ 0 ta có

P( z )
an −1 an −2  1
1 

1
+
+

+
...
+

n ÷
 z3
÷
zn
z

z2
z


> 1+
24

an −1 an −2
1
+ 2 − 2
.
z
z
z ( z − 1)


Nếu z thỏa mãn arg z ≤
Nếu z ≥

P( z )
1
π
1
>
1

2
thì Re  2 ÷ ≥ 0 suy ra
.
zn

z ( z − 1)
4
z 

3
2
thì z ( z − 1) > 1 nên P ( z ) ≠ 0
2

Nếu z là một nghiệm của P ( x) với arg z >

thì z <

π
. Giả sử Re z > 0 , khi đó theo bổ đề 1
4

1+ 5 3
1+ 5
, suy ra Re z <
< , bổ đề 2 được chứng minh.
2
2 2
2

Trở lại bài toán: Ta xét hai trường hợp:
* Nếu b ≥ 3 , theo bổ đề 1 thì nếu α là nghiệm của P ( x) thì b − α > 1 .
Giả sử P ( x) = f ( x).g ( x); deg f ≥ 1,deg g ≥ 1 . Từ P (b) nguyên tố suy ra f (b) = 1
hoặc g (b) = 1 . Ta xét f (b) = 1 , gọi các nghiệm của f là α1 ,α 2 ,...,α k .
Ta có f (b) = b − α1 b − α 2 ... b − α k > 1 , mâu thuẫn. Vậy P ( x) bất khả quy.

* Nếu b = 2 : Tương tự trên ta xét f (2) = 1 . Theo bổ để 2 thì mọi nghiệm của P ( x)
đều thỏa mãn Re z <

α1 ,α 2 ,...,α k

thì

3
, suy ra z − 2 > z − 1 . Do đó nếu gọi các nghiệm của f là
2

f (2) = 2 − α1 2 − α 2 ... 2 − α k > 1 − α1 1 − α 2 ...1 − α k = f (1) ≥ 1 ,

mâu thuẫn. Vậy P ( x) bất khả quy.
Nhận xét: Tiêu chuẩn Cohn's có một ý nghĩa thực tế là giúp ta tạo ra các đa thức bất
khả quy một cách dễ dàng.
Bài toán 11 (Hongkong TST 2011). Chứng minh rằng P ( x) = ( x 2 + x )

2n

+ 1 bất khả

quy trên ¢ [ x ] với mọi số tự nhiên n.
Lời giải. Giả sử P ( x) = G ( x).H ( x); G ( x), H ( x) ∈ ¢ [ x ] ;deg G,deg H ≥ 1 .
Xét trên trường ¢ 2 , ta có P ( x) = ( x 2 + x + 1) (mod 2) ⇒ ( x 2 + x + 1)
2n

với g ( x) ≡ G ( x); h( x) ≡ H ( x ) trên ¢ 2 .
25


2n

= g ( x).h( x )


Tài liệu bạn tìm kiếm đã sẵn sàng tải về

Tải bản đầy đủ ngay
×